Variation des Begriffs wie ∂RabRab∂Rabcd∂RabRab∂Rabcd\frac{\partial R_{ab} R^{ab}}{\partial R_{abcd}}, ∂RabcdRabcd∂Refgh∂RabcdRabcd∂Refgh\frac{\partial R_ {abcd} R^{abcd}}{\partial R_{efgh}}

Dies hängt mit meiner vorherigen Frage Variation bzgl. zusammen R A B C D ? Wie berechnet man R R A B C D = 1 2 ( G A C G B D G A D G B C ) ? In diesem Fall möchte ich den Ricci-Tensor berechnen

R A B R A B R A B C D =

Wie berechne ich in diesem Fall die Ableitungen?

Und weiter für

R A B C D R A B C D R e F G H
Kann ich frühere Derivate verwenden? ( X 2 ) ' = 2 X , und sagen Sie oben als 2 R e F G H ?

Antworten (2)

Wir beginnen damit, dies zu bemerken

R A B R C D e F = ( R C A e B G C e ) R C D e F = ( R C D e F δ A D δ B F G C e ) R C D e F ,
schlägt eine Antwort in der Art von vor δ A D δ B F G C e . Aber aufgrund der Antisymmetrieeigenschaften des Riemann-Tensors gibt es mehr als eine Schreibweise R A B als Widerspruch zu R C D e F mit einem Tensor.

Beim Austausch brauchen wir eine Antisymmetrie C mit D , was eine Antwort in der Art von vorschlägt 1 2 ( δ A D δ B F G C e δ A C δ B F G D e ) . Aber das kann auch nicht ganz stimmen: Wir brauchen auch eine Antisymmetrie beim Vertauschen e mit F , was eine Antwort in der Art von vorschlägt 1 4 ( δ A D δ B F G C e δ A C δ B F G D e δ A D δ B e G C F + δ A C δ B e G D F ) . Aber wir brauchen noch C D e F e F C D eine Symmetrie sein, die das Endergebnis liefert

R A B R C D e F = X A B C D e F := 1 8 ( ( δ A D δ B F + δ A F δ B D ) G C e ( δ A C δ B F + δ A F δ B C ) G D e ( δ A D δ B e + δ A e δ B D ) G C F + ( δ A C δ B e + δ A e δ B C ) G D F ) .

Beachten Sie, dass jeder Begriff hat A B als niedrigere Indizes und C D e F als obere Indizes.

Nach der Produktregel

( R A B R A B ) R C D e F = R A B R C D e F R A B + R A B R A B R C D e F .
Wir können die Höhen ändern A , B im zweiten Term, d.h.
( R A B R A B ) R C D e F = 2 R A B X A B C D e F .
Ausdrücke wie R A B δ A D δ B F G C e = G C e R D F geben
( R A B R A B ) R C D e F = 1 2 ( G C e R D F G D e R C F G C F R D e + G D F R C e ) .
Beachten Sie, dass jeder Begriff hat C D e F als obere Indizes und A B existieren auf der rechten Seite nicht, da es sich um auf der linken Seite herausgezogene Dummy-Indizes handelt.

Stellen Sie sich für die zweite Ableitung vor, wir wollten stattdessen ( v A v A ) v B für einen Vektor; Die Antwort wäre 2 v B , was eine Antwort wie vorschlägt 2 R e F G H . Dies hat bereits die richtigen Eigenschaften, also sind wir fertig.

Danke!. Ich habe einen weiteren Beitrag erstellt, physical.stackexchange.com/questions/301884/… . In dieser Zeit versuche ich Produkte zweier kontrahierter Riemann-Tensoren zu variieren. Wenn Sie daran interessiert sind, geben Sie bitte einen Kommentar ab.

Aus dem hilfreichen Kommentar @JG,

( R A B R A B ) R C D e F = 2 R A B X A B C D e F , X A B C D e F = R A B R C D e F

Versuchen Sie zu rechnen

R μ v = R A B C D G B D δ μ A δ v C = 1 8 R A B C D ( G B D δ μ A δ v C + G B D δ μ C δ v A G A D δ μ B δ v C G A D δ μ C δ v B G B C δ μ D δ v A G B C δ μ A δ v C + G A C δ μ D δ v B + G A C δ μ B δ v D )
wobei ich (a,b) und (c,d) antisymmetrisierte und die Paare (ab, cd) symmetrisierte und symmetrisierte μ , v in seitlichen Klammern

Also das vermute ich

X μ v A B C D = R μ v R A B C D = 1 8 ( G B D δ μ A δ v C + G B D δ μ C δ v A G A D δ μ B δ v C G A D δ μ C δ v B G B C δ μ D δ v A G B C δ μ A δ v C + G A C δ μ D δ v B + G A C δ μ B δ v D )

R μ v R μ v R A B C D = R A [ C G B ] D + R B [ D G A ] C = 1 2 ( R A C G B D R B C G A D R A D G C B + R B D G C A )

Habe ich recht?

Ich schreibe bald eine Antwort. Ihre Formel für X macht keinen Sinn; Es hat zwei Bonus-Indizes und platziert einige der Indizes auf der falschen Höhe.
@JG, Entschuldigung, es gibt einige Tippfehler. Ich korrigiere es
@JG, nachdem ich die Indizes richtig korrigiert habe, erhalte ich die gleiche Antwort in Ihrem Beitrag. Danke.